Can an uncountable group have a countable number of subgroups? [closed]











up vote
14
down vote

favorite
3













Can an uncountable group have only a countable number of subgroups?



Please give examples if any exist!




Edit: I want a group having uncountable cardinality but having a countable number of subgroups.



By countable number of subgroups, I mean the collection of all subgroups of a group is countable.










share|cite|improve this question















closed as off-topic by Shaun, Brian Borchers, user10354138, Cesareo, TheSimpliFire Dec 9 at 19:28


This question appears to be off-topic. The users who voted to close gave this specific reason:


  • "This question is missing context or other details: Please provide additional context, which ideally explains why the question is relevant to you and our community. Some forms of context include: background and motivation, relevant definitions, source, possible strategies, your current progress, why the question is interesting or important, etc." – Shaun, Brian Borchers, user10354138, Cesareo, TheSimpliFire

If this question can be reworded to fit the rules in the help center, please edit the question.









  • 5




    I'm frankly a bit surprised at the negative reaction to this question.
    – Noah Schweber
    Dec 9 at 3:00






  • 2




    Possible duplicate of Countable number of subgroups $implies $ countable group
    – Carmeister
    Dec 9 at 10:05










  • Then why is it marked as "off-topic" rather than "duplicate"?
    – C Monsour
    Dec 9 at 20:42










  • I'm voting to reopen. If it should be closed please give the correct reason.
    – C Monsour
    Dec 9 at 20:44










  • @CMonsour It was closed because there is no context. It's just a problem statement with no details. The closure reason is correct. While it might be better that it be listed as a duplicate, I don't see that as a reason to reopen.
    – jgon
    Dec 9 at 21:10















up vote
14
down vote

favorite
3













Can an uncountable group have only a countable number of subgroups?



Please give examples if any exist!




Edit: I want a group having uncountable cardinality but having a countable number of subgroups.



By countable number of subgroups, I mean the collection of all subgroups of a group is countable.










share|cite|improve this question















closed as off-topic by Shaun, Brian Borchers, user10354138, Cesareo, TheSimpliFire Dec 9 at 19:28


This question appears to be off-topic. The users who voted to close gave this specific reason:


  • "This question is missing context or other details: Please provide additional context, which ideally explains why the question is relevant to you and our community. Some forms of context include: background and motivation, relevant definitions, source, possible strategies, your current progress, why the question is interesting or important, etc." – Shaun, Brian Borchers, user10354138, Cesareo, TheSimpliFire

If this question can be reworded to fit the rules in the help center, please edit the question.









  • 5




    I'm frankly a bit surprised at the negative reaction to this question.
    – Noah Schweber
    Dec 9 at 3:00






  • 2




    Possible duplicate of Countable number of subgroups $implies $ countable group
    – Carmeister
    Dec 9 at 10:05










  • Then why is it marked as "off-topic" rather than "duplicate"?
    – C Monsour
    Dec 9 at 20:42










  • I'm voting to reopen. If it should be closed please give the correct reason.
    – C Monsour
    Dec 9 at 20:44










  • @CMonsour It was closed because there is no context. It's just a problem statement with no details. The closure reason is correct. While it might be better that it be listed as a duplicate, I don't see that as a reason to reopen.
    – jgon
    Dec 9 at 21:10













up vote
14
down vote

favorite
3









up vote
14
down vote

favorite
3






3






Can an uncountable group have only a countable number of subgroups?



Please give examples if any exist!




Edit: I want a group having uncountable cardinality but having a countable number of subgroups.



By countable number of subgroups, I mean the collection of all subgroups of a group is countable.










share|cite|improve this question
















Can an uncountable group have only a countable number of subgroups?



Please give examples if any exist!




Edit: I want a group having uncountable cardinality but having a countable number of subgroups.



By countable number of subgroups, I mean the collection of all subgroups of a group is countable.







group-theory examples-counterexamples infinite-groups






share|cite|improve this question















share|cite|improve this question













share|cite|improve this question




share|cite|improve this question








edited Dec 9 at 3:01









Shaun

8,215113578




8,215113578










asked Dec 9 at 2:47









Cloud JR

806417




806417




closed as off-topic by Shaun, Brian Borchers, user10354138, Cesareo, TheSimpliFire Dec 9 at 19:28


This question appears to be off-topic. The users who voted to close gave this specific reason:


  • "This question is missing context or other details: Please provide additional context, which ideally explains why the question is relevant to you and our community. Some forms of context include: background and motivation, relevant definitions, source, possible strategies, your current progress, why the question is interesting or important, etc." – Shaun, Brian Borchers, user10354138, Cesareo, TheSimpliFire

If this question can be reworded to fit the rules in the help center, please edit the question.




closed as off-topic by Shaun, Brian Borchers, user10354138, Cesareo, TheSimpliFire Dec 9 at 19:28


This question appears to be off-topic. The users who voted to close gave this specific reason:


  • "This question is missing context or other details: Please provide additional context, which ideally explains why the question is relevant to you and our community. Some forms of context include: background and motivation, relevant definitions, source, possible strategies, your current progress, why the question is interesting or important, etc." – Shaun, Brian Borchers, user10354138, Cesareo, TheSimpliFire

If this question can be reworded to fit the rules in the help center, please edit the question.








  • 5




    I'm frankly a bit surprised at the negative reaction to this question.
    – Noah Schweber
    Dec 9 at 3:00






  • 2




    Possible duplicate of Countable number of subgroups $implies $ countable group
    – Carmeister
    Dec 9 at 10:05










  • Then why is it marked as "off-topic" rather than "duplicate"?
    – C Monsour
    Dec 9 at 20:42










  • I'm voting to reopen. If it should be closed please give the correct reason.
    – C Monsour
    Dec 9 at 20:44










  • @CMonsour It was closed because there is no context. It's just a problem statement with no details. The closure reason is correct. While it might be better that it be listed as a duplicate, I don't see that as a reason to reopen.
    – jgon
    Dec 9 at 21:10














  • 5




    I'm frankly a bit surprised at the negative reaction to this question.
    – Noah Schweber
    Dec 9 at 3:00






  • 2




    Possible duplicate of Countable number of subgroups $implies $ countable group
    – Carmeister
    Dec 9 at 10:05










  • Then why is it marked as "off-topic" rather than "duplicate"?
    – C Monsour
    Dec 9 at 20:42










  • I'm voting to reopen. If it should be closed please give the correct reason.
    – C Monsour
    Dec 9 at 20:44










  • @CMonsour It was closed because there is no context. It's just a problem statement with no details. The closure reason is correct. While it might be better that it be listed as a duplicate, I don't see that as a reason to reopen.
    – jgon
    Dec 9 at 21:10








5




5




I'm frankly a bit surprised at the negative reaction to this question.
– Noah Schweber
Dec 9 at 3:00




I'm frankly a bit surprised at the negative reaction to this question.
– Noah Schweber
Dec 9 at 3:00




2




2




Possible duplicate of Countable number of subgroups $implies $ countable group
– Carmeister
Dec 9 at 10:05




Possible duplicate of Countable number of subgroups $implies $ countable group
– Carmeister
Dec 9 at 10:05












Then why is it marked as "off-topic" rather than "duplicate"?
– C Monsour
Dec 9 at 20:42




Then why is it marked as "off-topic" rather than "duplicate"?
– C Monsour
Dec 9 at 20:42












I'm voting to reopen. If it should be closed please give the correct reason.
– C Monsour
Dec 9 at 20:44




I'm voting to reopen. If it should be closed please give the correct reason.
– C Monsour
Dec 9 at 20:44












@CMonsour It was closed because there is no context. It's just a problem statement with no details. The closure reason is correct. While it might be better that it be listed as a duplicate, I don't see that as a reason to reopen.
– jgon
Dec 9 at 21:10




@CMonsour It was closed because there is no context. It's just a problem statement with no details. The closure reason is correct. While it might be better that it be listed as a duplicate, I don't see that as a reason to reopen.
– jgon
Dec 9 at 21:10










2 Answers
2






active

oldest

votes

















up vote
27
down vote



accepted










No. Suppose $G$ is an uncountable group. Every element $g$ of $G$ belongs to a countable subgroup of $G$, namely the cyclic subgroup $langle grangle$. Thus $G$ is the union of all of its countable subgroups. Since a countable union of countable sets is countable, $G$ must have uncountably many countable subgroups.






share|cite|improve this answer

















  • 1




    How can we be sure that the subgroups generated by $g$ and $g'$ (for $gnot= g'$) are not the same?
    – BenjaminH
    Dec 9 at 8:54












  • With a bit of refinement, this argument can be turned into the group being a countable union of finite sets. I forgot if this makes it countable without choice.
    – Tobias Kildetoft
    Dec 9 at 9:00










  • @TobiasKildetoft The axiom of choice is needed to prove that a countable union of two-element sets is countable. (If the union were countable, you could prove the axiom of choice for two-element sets.)
    – bof
    Dec 9 at 10:17






  • 4




    @BenjaminH : You can't, for a specific pair $g,g'$. However, both $langle grangle$ and $langle g'rangle$ are countable, so there are indeed uncountably many such subgroups (otherwise a countable union of countable sets would exhaust $G$, which is not possible).
    – MPW
    Dec 9 at 13:23


















up vote
9
down vote













EDIT: bof's answer is the right one, but the construction below - while completely pointless overkill - is still an example of a useful technique, so I'm leaving this answer up.





No, this cannot happen.



Suppose $G$ is a group and $A$ is a countable subset of $G$. Then the closure of $A$ under the group operations ($*$ and $^{-1}$) of $G$, $langle Arangle$, is again countable - this is a good exercise (HINT: the set of finite strings from a countable set is countable).



With this in hand, if $G$ is an uncountable group we can build an uncountable chain of subgroups of $G$, as follows:




  • We will define a countable subgroup $A_delta$ for every countable ordinal $delta$. There are uncountably many of these, so if we can do this we'll be done.


  • We let $A_0$ be the trivial subgroup.


  • Having defined $A_eta$ for every $eta<delta$, we let $a$ be some element of $G$ not in $bigcup_{eta<delta}A_eta$ - which exists, since this is a countable union of countable subgroups, and $G$ is uncountable - and let $A_delta=langle (bigcup_{eta<delta}A_eta)cup{a}rangle$.


  • It's easy to prove by transfinite induction that $(A_delta)_{delta<omega_1}$ is a strictly increasing chain of countable subgroups of $G$, so we're done.







share|cite|improve this answer





















  • Both answers use the axiom of choice. In ZF, can there be an uncountable group with only a countable number of subgroups?
    – bof
    Dec 9 at 3:43






  • 2




    @bof This question that I asked and Noah answered a while ago should answer your question
    – Paul Plummer
    Dec 9 at 4:51


















2 Answers
2






active

oldest

votes








2 Answers
2






active

oldest

votes









active

oldest

votes






active

oldest

votes








up vote
27
down vote



accepted










No. Suppose $G$ is an uncountable group. Every element $g$ of $G$ belongs to a countable subgroup of $G$, namely the cyclic subgroup $langle grangle$. Thus $G$ is the union of all of its countable subgroups. Since a countable union of countable sets is countable, $G$ must have uncountably many countable subgroups.






share|cite|improve this answer

















  • 1




    How can we be sure that the subgroups generated by $g$ and $g'$ (for $gnot= g'$) are not the same?
    – BenjaminH
    Dec 9 at 8:54












  • With a bit of refinement, this argument can be turned into the group being a countable union of finite sets. I forgot if this makes it countable without choice.
    – Tobias Kildetoft
    Dec 9 at 9:00










  • @TobiasKildetoft The axiom of choice is needed to prove that a countable union of two-element sets is countable. (If the union were countable, you could prove the axiom of choice for two-element sets.)
    – bof
    Dec 9 at 10:17






  • 4




    @BenjaminH : You can't, for a specific pair $g,g'$. However, both $langle grangle$ and $langle g'rangle$ are countable, so there are indeed uncountably many such subgroups (otherwise a countable union of countable sets would exhaust $G$, which is not possible).
    – MPW
    Dec 9 at 13:23















up vote
27
down vote



accepted










No. Suppose $G$ is an uncountable group. Every element $g$ of $G$ belongs to a countable subgroup of $G$, namely the cyclic subgroup $langle grangle$. Thus $G$ is the union of all of its countable subgroups. Since a countable union of countable sets is countable, $G$ must have uncountably many countable subgroups.






share|cite|improve this answer

















  • 1




    How can we be sure that the subgroups generated by $g$ and $g'$ (for $gnot= g'$) are not the same?
    – BenjaminH
    Dec 9 at 8:54












  • With a bit of refinement, this argument can be turned into the group being a countable union of finite sets. I forgot if this makes it countable without choice.
    – Tobias Kildetoft
    Dec 9 at 9:00










  • @TobiasKildetoft The axiom of choice is needed to prove that a countable union of two-element sets is countable. (If the union were countable, you could prove the axiom of choice for two-element sets.)
    – bof
    Dec 9 at 10:17






  • 4




    @BenjaminH : You can't, for a specific pair $g,g'$. However, both $langle grangle$ and $langle g'rangle$ are countable, so there are indeed uncountably many such subgroups (otherwise a countable union of countable sets would exhaust $G$, which is not possible).
    – MPW
    Dec 9 at 13:23













up vote
27
down vote



accepted







up vote
27
down vote



accepted






No. Suppose $G$ is an uncountable group. Every element $g$ of $G$ belongs to a countable subgroup of $G$, namely the cyclic subgroup $langle grangle$. Thus $G$ is the union of all of its countable subgroups. Since a countable union of countable sets is countable, $G$ must have uncountably many countable subgroups.






share|cite|improve this answer












No. Suppose $G$ is an uncountable group. Every element $g$ of $G$ belongs to a countable subgroup of $G$, namely the cyclic subgroup $langle grangle$. Thus $G$ is the union of all of its countable subgroups. Since a countable union of countable sets is countable, $G$ must have uncountably many countable subgroups.







share|cite|improve this answer












share|cite|improve this answer



share|cite|improve this answer










answered Dec 9 at 2:56









bof

49.9k457119




49.9k457119








  • 1




    How can we be sure that the subgroups generated by $g$ and $g'$ (for $gnot= g'$) are not the same?
    – BenjaminH
    Dec 9 at 8:54












  • With a bit of refinement, this argument can be turned into the group being a countable union of finite sets. I forgot if this makes it countable without choice.
    – Tobias Kildetoft
    Dec 9 at 9:00










  • @TobiasKildetoft The axiom of choice is needed to prove that a countable union of two-element sets is countable. (If the union were countable, you could prove the axiom of choice for two-element sets.)
    – bof
    Dec 9 at 10:17






  • 4




    @BenjaminH : You can't, for a specific pair $g,g'$. However, both $langle grangle$ and $langle g'rangle$ are countable, so there are indeed uncountably many such subgroups (otherwise a countable union of countable sets would exhaust $G$, which is not possible).
    – MPW
    Dec 9 at 13:23














  • 1




    How can we be sure that the subgroups generated by $g$ and $g'$ (for $gnot= g'$) are not the same?
    – BenjaminH
    Dec 9 at 8:54












  • With a bit of refinement, this argument can be turned into the group being a countable union of finite sets. I forgot if this makes it countable without choice.
    – Tobias Kildetoft
    Dec 9 at 9:00










  • @TobiasKildetoft The axiom of choice is needed to prove that a countable union of two-element sets is countable. (If the union were countable, you could prove the axiom of choice for two-element sets.)
    – bof
    Dec 9 at 10:17






  • 4




    @BenjaminH : You can't, for a specific pair $g,g'$. However, both $langle grangle$ and $langle g'rangle$ are countable, so there are indeed uncountably many such subgroups (otherwise a countable union of countable sets would exhaust $G$, which is not possible).
    – MPW
    Dec 9 at 13:23








1




1




How can we be sure that the subgroups generated by $g$ and $g'$ (for $gnot= g'$) are not the same?
– BenjaminH
Dec 9 at 8:54






How can we be sure that the subgroups generated by $g$ and $g'$ (for $gnot= g'$) are not the same?
– BenjaminH
Dec 9 at 8:54














With a bit of refinement, this argument can be turned into the group being a countable union of finite sets. I forgot if this makes it countable without choice.
– Tobias Kildetoft
Dec 9 at 9:00




With a bit of refinement, this argument can be turned into the group being a countable union of finite sets. I forgot if this makes it countable without choice.
– Tobias Kildetoft
Dec 9 at 9:00












@TobiasKildetoft The axiom of choice is needed to prove that a countable union of two-element sets is countable. (If the union were countable, you could prove the axiom of choice for two-element sets.)
– bof
Dec 9 at 10:17




@TobiasKildetoft The axiom of choice is needed to prove that a countable union of two-element sets is countable. (If the union were countable, you could prove the axiom of choice for two-element sets.)
– bof
Dec 9 at 10:17




4




4




@BenjaminH : You can't, for a specific pair $g,g'$. However, both $langle grangle$ and $langle g'rangle$ are countable, so there are indeed uncountably many such subgroups (otherwise a countable union of countable sets would exhaust $G$, which is not possible).
– MPW
Dec 9 at 13:23




@BenjaminH : You can't, for a specific pair $g,g'$. However, both $langle grangle$ and $langle g'rangle$ are countable, so there are indeed uncountably many such subgroups (otherwise a countable union of countable sets would exhaust $G$, which is not possible).
– MPW
Dec 9 at 13:23










up vote
9
down vote













EDIT: bof's answer is the right one, but the construction below - while completely pointless overkill - is still an example of a useful technique, so I'm leaving this answer up.





No, this cannot happen.



Suppose $G$ is a group and $A$ is a countable subset of $G$. Then the closure of $A$ under the group operations ($*$ and $^{-1}$) of $G$, $langle Arangle$, is again countable - this is a good exercise (HINT: the set of finite strings from a countable set is countable).



With this in hand, if $G$ is an uncountable group we can build an uncountable chain of subgroups of $G$, as follows:




  • We will define a countable subgroup $A_delta$ for every countable ordinal $delta$. There are uncountably many of these, so if we can do this we'll be done.


  • We let $A_0$ be the trivial subgroup.


  • Having defined $A_eta$ for every $eta<delta$, we let $a$ be some element of $G$ not in $bigcup_{eta<delta}A_eta$ - which exists, since this is a countable union of countable subgroups, and $G$ is uncountable - and let $A_delta=langle (bigcup_{eta<delta}A_eta)cup{a}rangle$.


  • It's easy to prove by transfinite induction that $(A_delta)_{delta<omega_1}$ is a strictly increasing chain of countable subgroups of $G$, so we're done.







share|cite|improve this answer





















  • Both answers use the axiom of choice. In ZF, can there be an uncountable group with only a countable number of subgroups?
    – bof
    Dec 9 at 3:43






  • 2




    @bof This question that I asked and Noah answered a while ago should answer your question
    – Paul Plummer
    Dec 9 at 4:51















up vote
9
down vote













EDIT: bof's answer is the right one, but the construction below - while completely pointless overkill - is still an example of a useful technique, so I'm leaving this answer up.





No, this cannot happen.



Suppose $G$ is a group and $A$ is a countable subset of $G$. Then the closure of $A$ under the group operations ($*$ and $^{-1}$) of $G$, $langle Arangle$, is again countable - this is a good exercise (HINT: the set of finite strings from a countable set is countable).



With this in hand, if $G$ is an uncountable group we can build an uncountable chain of subgroups of $G$, as follows:




  • We will define a countable subgroup $A_delta$ for every countable ordinal $delta$. There are uncountably many of these, so if we can do this we'll be done.


  • We let $A_0$ be the trivial subgroup.


  • Having defined $A_eta$ for every $eta<delta$, we let $a$ be some element of $G$ not in $bigcup_{eta<delta}A_eta$ - which exists, since this is a countable union of countable subgroups, and $G$ is uncountable - and let $A_delta=langle (bigcup_{eta<delta}A_eta)cup{a}rangle$.


  • It's easy to prove by transfinite induction that $(A_delta)_{delta<omega_1}$ is a strictly increasing chain of countable subgroups of $G$, so we're done.







share|cite|improve this answer





















  • Both answers use the axiom of choice. In ZF, can there be an uncountable group with only a countable number of subgroups?
    – bof
    Dec 9 at 3:43






  • 2




    @bof This question that I asked and Noah answered a while ago should answer your question
    – Paul Plummer
    Dec 9 at 4:51













up vote
9
down vote










up vote
9
down vote









EDIT: bof's answer is the right one, but the construction below - while completely pointless overkill - is still an example of a useful technique, so I'm leaving this answer up.





No, this cannot happen.



Suppose $G$ is a group and $A$ is a countable subset of $G$. Then the closure of $A$ under the group operations ($*$ and $^{-1}$) of $G$, $langle Arangle$, is again countable - this is a good exercise (HINT: the set of finite strings from a countable set is countable).



With this in hand, if $G$ is an uncountable group we can build an uncountable chain of subgroups of $G$, as follows:




  • We will define a countable subgroup $A_delta$ for every countable ordinal $delta$. There are uncountably many of these, so if we can do this we'll be done.


  • We let $A_0$ be the trivial subgroup.


  • Having defined $A_eta$ for every $eta<delta$, we let $a$ be some element of $G$ not in $bigcup_{eta<delta}A_eta$ - which exists, since this is a countable union of countable subgroups, and $G$ is uncountable - and let $A_delta=langle (bigcup_{eta<delta}A_eta)cup{a}rangle$.


  • It's easy to prove by transfinite induction that $(A_delta)_{delta<omega_1}$ is a strictly increasing chain of countable subgroups of $G$, so we're done.







share|cite|improve this answer












EDIT: bof's answer is the right one, but the construction below - while completely pointless overkill - is still an example of a useful technique, so I'm leaving this answer up.





No, this cannot happen.



Suppose $G$ is a group and $A$ is a countable subset of $G$. Then the closure of $A$ under the group operations ($*$ and $^{-1}$) of $G$, $langle Arangle$, is again countable - this is a good exercise (HINT: the set of finite strings from a countable set is countable).



With this in hand, if $G$ is an uncountable group we can build an uncountable chain of subgroups of $G$, as follows:




  • We will define a countable subgroup $A_delta$ for every countable ordinal $delta$. There are uncountably many of these, so if we can do this we'll be done.


  • We let $A_0$ be the trivial subgroup.


  • Having defined $A_eta$ for every $eta<delta$, we let $a$ be some element of $G$ not in $bigcup_{eta<delta}A_eta$ - which exists, since this is a countable union of countable subgroups, and $G$ is uncountable - and let $A_delta=langle (bigcup_{eta<delta}A_eta)cup{a}rangle$.


  • It's easy to prove by transfinite induction that $(A_delta)_{delta<omega_1}$ is a strictly increasing chain of countable subgroups of $G$, so we're done.








share|cite|improve this answer












share|cite|improve this answer



share|cite|improve this answer










answered Dec 9 at 2:58









Noah Schweber

120k10146278




120k10146278












  • Both answers use the axiom of choice. In ZF, can there be an uncountable group with only a countable number of subgroups?
    – bof
    Dec 9 at 3:43






  • 2




    @bof This question that I asked and Noah answered a while ago should answer your question
    – Paul Plummer
    Dec 9 at 4:51


















  • Both answers use the axiom of choice. In ZF, can there be an uncountable group with only a countable number of subgroups?
    – bof
    Dec 9 at 3:43






  • 2




    @bof This question that I asked and Noah answered a while ago should answer your question
    – Paul Plummer
    Dec 9 at 4:51
















Both answers use the axiom of choice. In ZF, can there be an uncountable group with only a countable number of subgroups?
– bof
Dec 9 at 3:43




Both answers use the axiom of choice. In ZF, can there be an uncountable group with only a countable number of subgroups?
– bof
Dec 9 at 3:43




2




2




@bof This question that I asked and Noah answered a while ago should answer your question
– Paul Plummer
Dec 9 at 4:51




@bof This question that I asked and Noah answered a while ago should answer your question
– Paul Plummer
Dec 9 at 4:51



Popular posts from this blog

How do I know what Microsoft account the skydrive app is syncing to?

When does type information flow backwards in C++?

Grease: Live!